LSAT and Law School Admissions Forum

Get expert LSAT preparation and law school admissions advice from PowerScore Test Preparation.

User avatar
 Dave Killoran
PowerScore Staff
  • PowerScore Staff
  • Posts: 5850
  • Joined: Mar 25, 2011
|
#88199
Complete Question Explanation
(The complete setup for this game can be found here: lsat/viewtopic.php?f=171&p=88197#p88197)

The correct answer choice is (B)

If L is third, then M must be fourth. From the third rule, we can then deduce that O is fifth and S is sixth. R and V must then be first and second, not necessarily in that order:

G3-Q16-d1.png

Accordingly, answer choice (B) is correct.
 deck1134
  • Posts: 160
  • Joined: Jun 11, 2018
|
#49062
Hi PowerScore,

If L is third, I drew out:

S
O
M
L
_
_
How is it possible that V goes to 5? There is not any space left. Thanks!
 Adam Tyson
PowerScore Staff
  • PowerScore Staff
  • Posts: 5153
  • Joined: Apr 14, 2011
|
#49113
You're right, Deck, and your diagram is perfect! V and R are interchangeable in the bottom two spaces, 1 and 2. That makes B the correct answer here - V can be directly above R, as follows:

S
O
M
L
V
R

It looks to me like you are double-checking answer A, which is not the correct answer, as you proved. C is also impossible, because O is immediately above M. D is impossible R is at least two layers lower than M. D is impossible because S must be the top layer.

B could be true, and so B is the right answer. If you answer key says otherwise, it's wrong, or perhaps you are looking at the wrong section in the key.

Good work!

Get the most out of your LSAT Prep Plus subscription.

Analyze and track your performance with our Testing and Analytics Package.